A proof that $sqrt{2}$ is not a rational number.












5












$begingroup$



Is this proof correct?




Suppose that $sqrt{2}=frac{a}{b}$, where $a,b in mathbb{N}$ and $a$ is as small as possible. Then $sqrt{2}b=a$ which means $2b=sqrt{2} a$. So we rewrite $sqrt{2}=frac{a}{b}cdotfrac{sqrt{2}-1}{sqrt{2}-1}=frac{sqrt{2}a-a}{sqrt{2}b-b}=frac{2b-a}{a-b}.,$ Note $,2b-a=a(sqrt{2}-1)<a$. So this fraction has a smaller numerator than the one we had. So this is a contradiction.










share|cite|improve this question











$endgroup$








  • 10




    $begingroup$
    Yes, this is a well-known proof.
    $endgroup$
    – Lord Shark the Unknown
    Dec 8 '18 at 13:15






  • 3




    $begingroup$
    @user42493 This is essentially the same as this section with $a=m$, $b=n$, $k=2$, $q=1$. At least you were creative enough to 'discover' another proof.
    $endgroup$
    – Toby Mak
    Dec 8 '18 at 13:18












  • $begingroup$
    You have a typographical error. ab-a should read $2b-a$.
    $endgroup$
    – Ben W
    Dec 8 '18 at 13:44










  • $begingroup$
    Note that $2b-a<a$ is equivalent to $2b<2a$, which is true because $a>b$.
    $endgroup$
    – egreg
    Dec 8 '18 at 13:57
















5












$begingroup$



Is this proof correct?




Suppose that $sqrt{2}=frac{a}{b}$, where $a,b in mathbb{N}$ and $a$ is as small as possible. Then $sqrt{2}b=a$ which means $2b=sqrt{2} a$. So we rewrite $sqrt{2}=frac{a}{b}cdotfrac{sqrt{2}-1}{sqrt{2}-1}=frac{sqrt{2}a-a}{sqrt{2}b-b}=frac{2b-a}{a-b}.,$ Note $,2b-a=a(sqrt{2}-1)<a$. So this fraction has a smaller numerator than the one we had. So this is a contradiction.










share|cite|improve this question











$endgroup$








  • 10




    $begingroup$
    Yes, this is a well-known proof.
    $endgroup$
    – Lord Shark the Unknown
    Dec 8 '18 at 13:15






  • 3




    $begingroup$
    @user42493 This is essentially the same as this section with $a=m$, $b=n$, $k=2$, $q=1$. At least you were creative enough to 'discover' another proof.
    $endgroup$
    – Toby Mak
    Dec 8 '18 at 13:18












  • $begingroup$
    You have a typographical error. ab-a should read $2b-a$.
    $endgroup$
    – Ben W
    Dec 8 '18 at 13:44










  • $begingroup$
    Note that $2b-a<a$ is equivalent to $2b<2a$, which is true because $a>b$.
    $endgroup$
    – egreg
    Dec 8 '18 at 13:57














5












5








5


1



$begingroup$



Is this proof correct?




Suppose that $sqrt{2}=frac{a}{b}$, where $a,b in mathbb{N}$ and $a$ is as small as possible. Then $sqrt{2}b=a$ which means $2b=sqrt{2} a$. So we rewrite $sqrt{2}=frac{a}{b}cdotfrac{sqrt{2}-1}{sqrt{2}-1}=frac{sqrt{2}a-a}{sqrt{2}b-b}=frac{2b-a}{a-b}.,$ Note $,2b-a=a(sqrt{2}-1)<a$. So this fraction has a smaller numerator than the one we had. So this is a contradiction.










share|cite|improve this question











$endgroup$





Is this proof correct?




Suppose that $sqrt{2}=frac{a}{b}$, where $a,b in mathbb{N}$ and $a$ is as small as possible. Then $sqrt{2}b=a$ which means $2b=sqrt{2} a$. So we rewrite $sqrt{2}=frac{a}{b}cdotfrac{sqrt{2}-1}{sqrt{2}-1}=frac{sqrt{2}a-a}{sqrt{2}b-b}=frac{2b-a}{a-b}.,$ Note $,2b-a=a(sqrt{2}-1)<a$. So this fraction has a smaller numerator than the one we had. So this is a contradiction.







elementary-number-theory proof-verification rationality-testing






share|cite|improve this question















share|cite|improve this question













share|cite|improve this question




share|cite|improve this question








edited Dec 8 '18 at 17:00









Bill Dubuque

209k29191639




209k29191639










asked Dec 8 '18 at 13:13









user42493user42493

1837




1837








  • 10




    $begingroup$
    Yes, this is a well-known proof.
    $endgroup$
    – Lord Shark the Unknown
    Dec 8 '18 at 13:15






  • 3




    $begingroup$
    @user42493 This is essentially the same as this section with $a=m$, $b=n$, $k=2$, $q=1$. At least you were creative enough to 'discover' another proof.
    $endgroup$
    – Toby Mak
    Dec 8 '18 at 13:18












  • $begingroup$
    You have a typographical error. ab-a should read $2b-a$.
    $endgroup$
    – Ben W
    Dec 8 '18 at 13:44










  • $begingroup$
    Note that $2b-a<a$ is equivalent to $2b<2a$, which is true because $a>b$.
    $endgroup$
    – egreg
    Dec 8 '18 at 13:57














  • 10




    $begingroup$
    Yes, this is a well-known proof.
    $endgroup$
    – Lord Shark the Unknown
    Dec 8 '18 at 13:15






  • 3




    $begingroup$
    @user42493 This is essentially the same as this section with $a=m$, $b=n$, $k=2$, $q=1$. At least you were creative enough to 'discover' another proof.
    $endgroup$
    – Toby Mak
    Dec 8 '18 at 13:18












  • $begingroup$
    You have a typographical error. ab-a should read $2b-a$.
    $endgroup$
    – Ben W
    Dec 8 '18 at 13:44










  • $begingroup$
    Note that $2b-a<a$ is equivalent to $2b<2a$, which is true because $a>b$.
    $endgroup$
    – egreg
    Dec 8 '18 at 13:57








10




10




$begingroup$
Yes, this is a well-known proof.
$endgroup$
– Lord Shark the Unknown
Dec 8 '18 at 13:15




$begingroup$
Yes, this is a well-known proof.
$endgroup$
– Lord Shark the Unknown
Dec 8 '18 at 13:15




3




3




$begingroup$
@user42493 This is essentially the same as this section with $a=m$, $b=n$, $k=2$, $q=1$. At least you were creative enough to 'discover' another proof.
$endgroup$
– Toby Mak
Dec 8 '18 at 13:18






$begingroup$
@user42493 This is essentially the same as this section with $a=m$, $b=n$, $k=2$, $q=1$. At least you were creative enough to 'discover' another proof.
$endgroup$
– Toby Mak
Dec 8 '18 at 13:18














$begingroup$
You have a typographical error. ab-a should read $2b-a$.
$endgroup$
– Ben W
Dec 8 '18 at 13:44




$begingroup$
You have a typographical error. ab-a should read $2b-a$.
$endgroup$
– Ben W
Dec 8 '18 at 13:44












$begingroup$
Note that $2b-a<a$ is equivalent to $2b<2a$, which is true because $a>b$.
$endgroup$
– egreg
Dec 8 '18 at 13:57




$begingroup$
Note that $2b-a<a$ is equivalent to $2b<2a$, which is true because $a>b$.
$endgroup$
– egreg
Dec 8 '18 at 13:57










1 Answer
1






active

oldest

votes


















0












$begingroup$

It is a correct well-known proof. Essentially it uses denominator descent by the division algorithm, though that is obfuscated . Below I clarify this viewpoint for the generalization below. The proof in the question is exactly the special case $, k = 2,$ and $,q = {rm floor}(sqrt 2) = 1,$ of the proof below.




Irrationality of $sqrt k,$ if it is not an integer (excerpted from Wikipedia, slightly edited)



For an integer $k>0$, suppose $sqrt k$ is not an integer, but is rational and can be expressed as $frac{a}b$ for natural numbers $a$ and $b$, and let $q$ be the largest integer no greater than $sqrt k.,$ Then



begin{aligned}{sqrt {k}}&={frac {a}{b}}\[8pt]&={frac {a({sqrt {k}}-q)}{b({sqrt {k}}-q)}}\[8pt]&={frac {a{sqrt {k}}-aq}{b{sqrt {k}}-bq}}\[8pt]&={frac {(b{sqrt {k}}){sqrt {k}}-aq}{b({frac {a}{b}})-bq}}\[8pt]&={frac {bk-aq}{a-bq}}end{aligned}



The numerator and denominator were each multiplied by $(sqrt k − q),$ — which is positive but less than $1$ and then simplified independently. So the two resulting products, say $a'$ and $b'$, are themselves integers, which are less than $a$ and $b$ respectively. Therefore, no matter what natural numbers $a$ and $b$ are used to express $sqrt k$, there exist smaller natural numbers $a' < a$ and $b' < b$ that have the same ratio. But infinite descent on the natural numbers is impossible, so this disproves the original assumption that $sqrt k$ could be expressed as a ratio of natural numbers.




We can rewrite the above proof more conceptually as below, where "$,n,$ is a denom of $,r$" means that the rational $,r,$ can be written with denominator $,n,,$ i.e. $,n,r = j,$ for some integer $,j.$



$begin{align}
[![1]!]qquadqquad, b sqrt k, &=, aqquad , Rightarrow,qquad text{$,b,$ is a denom of } sqrt k\
sqrt k,cdot, [![1]!] , Rightarrow,[![2]!]qquadqquad a sqrt k, &=, bkqquad Rightarrowqquad, text{ $a,$ is a denom of } sqrt k\
[![2]!] - [![1]!]q,Rightarrow,[![3]!] , (color{#c00}{a!-!bq})sqrt k, &=, bk!-!aq,Rightarrow, color{#c00}{abmod b} , text{ is a denom of } sqrt k\
end{align}$



If $,b,$ doesn't divide $,a,$ we get a smaller denom $, 0 < color{#c00}{a bmod b} < b,$ so infinite descent (on denoms), contra $Bbb N,$ is well-ordered. Hence $,b,$ divides $,a,,$ so $,sqrt k = a/b = nin Bbb Z,,$ so $,k = n^2$.



Alternatively we can initially assume that $,b,$ is the least denominator then deduce a contradiction that a smaller denominator exists if $,b,$ doesn't divide $,a.$



This method generalizes to show the $,Bbb Z,$ (or any PID) is integrally-closed, i.e. no proper fraction is a root of a polynomial that is monic (lead coef $= 1),,$ i.e. the monic case of the Rational Root Test. You can find much further discussion of this and related ideas in my posts on denominator ideals.






share|cite|improve this answer











$endgroup$













    Your Answer





    StackExchange.ifUsing("editor", function () {
    return StackExchange.using("mathjaxEditing", function () {
    StackExchange.MarkdownEditor.creationCallbacks.add(function (editor, postfix) {
    StackExchange.mathjaxEditing.prepareWmdForMathJax(editor, postfix, [["$", "$"], ["\\(","\\)"]]);
    });
    });
    }, "mathjax-editing");

    StackExchange.ready(function() {
    var channelOptions = {
    tags: "".split(" "),
    id: "69"
    };
    initTagRenderer("".split(" "), "".split(" "), channelOptions);

    StackExchange.using("externalEditor", function() {
    // Have to fire editor after snippets, if snippets enabled
    if (StackExchange.settings.snippets.snippetsEnabled) {
    StackExchange.using("snippets", function() {
    createEditor();
    });
    }
    else {
    createEditor();
    }
    });

    function createEditor() {
    StackExchange.prepareEditor({
    heartbeatType: 'answer',
    autoActivateHeartbeat: false,
    convertImagesToLinks: true,
    noModals: true,
    showLowRepImageUploadWarning: true,
    reputationToPostImages: 10,
    bindNavPrevention: true,
    postfix: "",
    imageUploader: {
    brandingHtml: "Powered by u003ca class="icon-imgur-white" href="https://imgur.com/"u003eu003c/au003e",
    contentPolicyHtml: "User contributions licensed under u003ca href="https://creativecommons.org/licenses/by-sa/3.0/"u003ecc by-sa 3.0 with attribution requiredu003c/au003e u003ca href="https://stackoverflow.com/legal/content-policy"u003e(content policy)u003c/au003e",
    allowUrls: true
    },
    noCode: true, onDemand: true,
    discardSelector: ".discard-answer"
    ,immediatelyShowMarkdownHelp:true
    });


    }
    });














    draft saved

    draft discarded


















    StackExchange.ready(
    function () {
    StackExchange.openid.initPostLogin('.new-post-login', 'https%3a%2f%2fmath.stackexchange.com%2fquestions%2f3031094%2fa-proof-that-sqrt2-is-not-a-rational-number%23new-answer', 'question_page');
    }
    );

    Post as a guest















    Required, but never shown

























    1 Answer
    1






    active

    oldest

    votes








    1 Answer
    1






    active

    oldest

    votes









    active

    oldest

    votes






    active

    oldest

    votes









    0












    $begingroup$

    It is a correct well-known proof. Essentially it uses denominator descent by the division algorithm, though that is obfuscated . Below I clarify this viewpoint for the generalization below. The proof in the question is exactly the special case $, k = 2,$ and $,q = {rm floor}(sqrt 2) = 1,$ of the proof below.




    Irrationality of $sqrt k,$ if it is not an integer (excerpted from Wikipedia, slightly edited)



    For an integer $k>0$, suppose $sqrt k$ is not an integer, but is rational and can be expressed as $frac{a}b$ for natural numbers $a$ and $b$, and let $q$ be the largest integer no greater than $sqrt k.,$ Then



    begin{aligned}{sqrt {k}}&={frac {a}{b}}\[8pt]&={frac {a({sqrt {k}}-q)}{b({sqrt {k}}-q)}}\[8pt]&={frac {a{sqrt {k}}-aq}{b{sqrt {k}}-bq}}\[8pt]&={frac {(b{sqrt {k}}){sqrt {k}}-aq}{b({frac {a}{b}})-bq}}\[8pt]&={frac {bk-aq}{a-bq}}end{aligned}



    The numerator and denominator were each multiplied by $(sqrt k − q),$ — which is positive but less than $1$ and then simplified independently. So the two resulting products, say $a'$ and $b'$, are themselves integers, which are less than $a$ and $b$ respectively. Therefore, no matter what natural numbers $a$ and $b$ are used to express $sqrt k$, there exist smaller natural numbers $a' < a$ and $b' < b$ that have the same ratio. But infinite descent on the natural numbers is impossible, so this disproves the original assumption that $sqrt k$ could be expressed as a ratio of natural numbers.




    We can rewrite the above proof more conceptually as below, where "$,n,$ is a denom of $,r$" means that the rational $,r,$ can be written with denominator $,n,,$ i.e. $,n,r = j,$ for some integer $,j.$



    $begin{align}
    [![1]!]qquadqquad, b sqrt k, &=, aqquad , Rightarrow,qquad text{$,b,$ is a denom of } sqrt k\
    sqrt k,cdot, [![1]!] , Rightarrow,[![2]!]qquadqquad a sqrt k, &=, bkqquad Rightarrowqquad, text{ $a,$ is a denom of } sqrt k\
    [![2]!] - [![1]!]q,Rightarrow,[![3]!] , (color{#c00}{a!-!bq})sqrt k, &=, bk!-!aq,Rightarrow, color{#c00}{abmod b} , text{ is a denom of } sqrt k\
    end{align}$



    If $,b,$ doesn't divide $,a,$ we get a smaller denom $, 0 < color{#c00}{a bmod b} < b,$ so infinite descent (on denoms), contra $Bbb N,$ is well-ordered. Hence $,b,$ divides $,a,,$ so $,sqrt k = a/b = nin Bbb Z,,$ so $,k = n^2$.



    Alternatively we can initially assume that $,b,$ is the least denominator then deduce a contradiction that a smaller denominator exists if $,b,$ doesn't divide $,a.$



    This method generalizes to show the $,Bbb Z,$ (or any PID) is integrally-closed, i.e. no proper fraction is a root of a polynomial that is monic (lead coef $= 1),,$ i.e. the monic case of the Rational Root Test. You can find much further discussion of this and related ideas in my posts on denominator ideals.






    share|cite|improve this answer











    $endgroup$


















      0












      $begingroup$

      It is a correct well-known proof. Essentially it uses denominator descent by the division algorithm, though that is obfuscated . Below I clarify this viewpoint for the generalization below. The proof in the question is exactly the special case $, k = 2,$ and $,q = {rm floor}(sqrt 2) = 1,$ of the proof below.




      Irrationality of $sqrt k,$ if it is not an integer (excerpted from Wikipedia, slightly edited)



      For an integer $k>0$, suppose $sqrt k$ is not an integer, but is rational and can be expressed as $frac{a}b$ for natural numbers $a$ and $b$, and let $q$ be the largest integer no greater than $sqrt k.,$ Then



      begin{aligned}{sqrt {k}}&={frac {a}{b}}\[8pt]&={frac {a({sqrt {k}}-q)}{b({sqrt {k}}-q)}}\[8pt]&={frac {a{sqrt {k}}-aq}{b{sqrt {k}}-bq}}\[8pt]&={frac {(b{sqrt {k}}){sqrt {k}}-aq}{b({frac {a}{b}})-bq}}\[8pt]&={frac {bk-aq}{a-bq}}end{aligned}



      The numerator and denominator were each multiplied by $(sqrt k − q),$ — which is positive but less than $1$ and then simplified independently. So the two resulting products, say $a'$ and $b'$, are themselves integers, which are less than $a$ and $b$ respectively. Therefore, no matter what natural numbers $a$ and $b$ are used to express $sqrt k$, there exist smaller natural numbers $a' < a$ and $b' < b$ that have the same ratio. But infinite descent on the natural numbers is impossible, so this disproves the original assumption that $sqrt k$ could be expressed as a ratio of natural numbers.




      We can rewrite the above proof more conceptually as below, where "$,n,$ is a denom of $,r$" means that the rational $,r,$ can be written with denominator $,n,,$ i.e. $,n,r = j,$ for some integer $,j.$



      $begin{align}
      [![1]!]qquadqquad, b sqrt k, &=, aqquad , Rightarrow,qquad text{$,b,$ is a denom of } sqrt k\
      sqrt k,cdot, [![1]!] , Rightarrow,[![2]!]qquadqquad a sqrt k, &=, bkqquad Rightarrowqquad, text{ $a,$ is a denom of } sqrt k\
      [![2]!] - [![1]!]q,Rightarrow,[![3]!] , (color{#c00}{a!-!bq})sqrt k, &=, bk!-!aq,Rightarrow, color{#c00}{abmod b} , text{ is a denom of } sqrt k\
      end{align}$



      If $,b,$ doesn't divide $,a,$ we get a smaller denom $, 0 < color{#c00}{a bmod b} < b,$ so infinite descent (on denoms), contra $Bbb N,$ is well-ordered. Hence $,b,$ divides $,a,,$ so $,sqrt k = a/b = nin Bbb Z,,$ so $,k = n^2$.



      Alternatively we can initially assume that $,b,$ is the least denominator then deduce a contradiction that a smaller denominator exists if $,b,$ doesn't divide $,a.$



      This method generalizes to show the $,Bbb Z,$ (or any PID) is integrally-closed, i.e. no proper fraction is a root of a polynomial that is monic (lead coef $= 1),,$ i.e. the monic case of the Rational Root Test. You can find much further discussion of this and related ideas in my posts on denominator ideals.






      share|cite|improve this answer











      $endgroup$
















        0












        0








        0





        $begingroup$

        It is a correct well-known proof. Essentially it uses denominator descent by the division algorithm, though that is obfuscated . Below I clarify this viewpoint for the generalization below. The proof in the question is exactly the special case $, k = 2,$ and $,q = {rm floor}(sqrt 2) = 1,$ of the proof below.




        Irrationality of $sqrt k,$ if it is not an integer (excerpted from Wikipedia, slightly edited)



        For an integer $k>0$, suppose $sqrt k$ is not an integer, but is rational and can be expressed as $frac{a}b$ for natural numbers $a$ and $b$, and let $q$ be the largest integer no greater than $sqrt k.,$ Then



        begin{aligned}{sqrt {k}}&={frac {a}{b}}\[8pt]&={frac {a({sqrt {k}}-q)}{b({sqrt {k}}-q)}}\[8pt]&={frac {a{sqrt {k}}-aq}{b{sqrt {k}}-bq}}\[8pt]&={frac {(b{sqrt {k}}){sqrt {k}}-aq}{b({frac {a}{b}})-bq}}\[8pt]&={frac {bk-aq}{a-bq}}end{aligned}



        The numerator and denominator were each multiplied by $(sqrt k − q),$ — which is positive but less than $1$ and then simplified independently. So the two resulting products, say $a'$ and $b'$, are themselves integers, which are less than $a$ and $b$ respectively. Therefore, no matter what natural numbers $a$ and $b$ are used to express $sqrt k$, there exist smaller natural numbers $a' < a$ and $b' < b$ that have the same ratio. But infinite descent on the natural numbers is impossible, so this disproves the original assumption that $sqrt k$ could be expressed as a ratio of natural numbers.




        We can rewrite the above proof more conceptually as below, where "$,n,$ is a denom of $,r$" means that the rational $,r,$ can be written with denominator $,n,,$ i.e. $,n,r = j,$ for some integer $,j.$



        $begin{align}
        [![1]!]qquadqquad, b sqrt k, &=, aqquad , Rightarrow,qquad text{$,b,$ is a denom of } sqrt k\
        sqrt k,cdot, [![1]!] , Rightarrow,[![2]!]qquadqquad a sqrt k, &=, bkqquad Rightarrowqquad, text{ $a,$ is a denom of } sqrt k\
        [![2]!] - [![1]!]q,Rightarrow,[![3]!] , (color{#c00}{a!-!bq})sqrt k, &=, bk!-!aq,Rightarrow, color{#c00}{abmod b} , text{ is a denom of } sqrt k\
        end{align}$



        If $,b,$ doesn't divide $,a,$ we get a smaller denom $, 0 < color{#c00}{a bmod b} < b,$ so infinite descent (on denoms), contra $Bbb N,$ is well-ordered. Hence $,b,$ divides $,a,,$ so $,sqrt k = a/b = nin Bbb Z,,$ so $,k = n^2$.



        Alternatively we can initially assume that $,b,$ is the least denominator then deduce a contradiction that a smaller denominator exists if $,b,$ doesn't divide $,a.$



        This method generalizes to show the $,Bbb Z,$ (or any PID) is integrally-closed, i.e. no proper fraction is a root of a polynomial that is monic (lead coef $= 1),,$ i.e. the monic case of the Rational Root Test. You can find much further discussion of this and related ideas in my posts on denominator ideals.






        share|cite|improve this answer











        $endgroup$



        It is a correct well-known proof. Essentially it uses denominator descent by the division algorithm, though that is obfuscated . Below I clarify this viewpoint for the generalization below. The proof in the question is exactly the special case $, k = 2,$ and $,q = {rm floor}(sqrt 2) = 1,$ of the proof below.




        Irrationality of $sqrt k,$ if it is not an integer (excerpted from Wikipedia, slightly edited)



        For an integer $k>0$, suppose $sqrt k$ is not an integer, but is rational and can be expressed as $frac{a}b$ for natural numbers $a$ and $b$, and let $q$ be the largest integer no greater than $sqrt k.,$ Then



        begin{aligned}{sqrt {k}}&={frac {a}{b}}\[8pt]&={frac {a({sqrt {k}}-q)}{b({sqrt {k}}-q)}}\[8pt]&={frac {a{sqrt {k}}-aq}{b{sqrt {k}}-bq}}\[8pt]&={frac {(b{sqrt {k}}){sqrt {k}}-aq}{b({frac {a}{b}})-bq}}\[8pt]&={frac {bk-aq}{a-bq}}end{aligned}



        The numerator and denominator were each multiplied by $(sqrt k − q),$ — which is positive but less than $1$ and then simplified independently. So the two resulting products, say $a'$ and $b'$, are themselves integers, which are less than $a$ and $b$ respectively. Therefore, no matter what natural numbers $a$ and $b$ are used to express $sqrt k$, there exist smaller natural numbers $a' < a$ and $b' < b$ that have the same ratio. But infinite descent on the natural numbers is impossible, so this disproves the original assumption that $sqrt k$ could be expressed as a ratio of natural numbers.




        We can rewrite the above proof more conceptually as below, where "$,n,$ is a denom of $,r$" means that the rational $,r,$ can be written with denominator $,n,,$ i.e. $,n,r = j,$ for some integer $,j.$



        $begin{align}
        [![1]!]qquadqquad, b sqrt k, &=, aqquad , Rightarrow,qquad text{$,b,$ is a denom of } sqrt k\
        sqrt k,cdot, [![1]!] , Rightarrow,[![2]!]qquadqquad a sqrt k, &=, bkqquad Rightarrowqquad, text{ $a,$ is a denom of } sqrt k\
        [![2]!] - [![1]!]q,Rightarrow,[![3]!] , (color{#c00}{a!-!bq})sqrt k, &=, bk!-!aq,Rightarrow, color{#c00}{abmod b} , text{ is a denom of } sqrt k\
        end{align}$



        If $,b,$ doesn't divide $,a,$ we get a smaller denom $, 0 < color{#c00}{a bmod b} < b,$ so infinite descent (on denoms), contra $Bbb N,$ is well-ordered. Hence $,b,$ divides $,a,,$ so $,sqrt k = a/b = nin Bbb Z,,$ so $,k = n^2$.



        Alternatively we can initially assume that $,b,$ is the least denominator then deduce a contradiction that a smaller denominator exists if $,b,$ doesn't divide $,a.$



        This method generalizes to show the $,Bbb Z,$ (or any PID) is integrally-closed, i.e. no proper fraction is a root of a polynomial that is monic (lead coef $= 1),,$ i.e. the monic case of the Rational Root Test. You can find much further discussion of this and related ideas in my posts on denominator ideals.







        share|cite|improve this answer














        share|cite|improve this answer



        share|cite|improve this answer








        edited Dec 8 '18 at 16:52

























        answered Dec 8 '18 at 15:36









        Bill DubuqueBill Dubuque

        209k29191639




        209k29191639






























            draft saved

            draft discarded




















































            Thanks for contributing an answer to Mathematics Stack Exchange!


            • Please be sure to answer the question. Provide details and share your research!

            But avoid



            • Asking for help, clarification, or responding to other answers.

            • Making statements based on opinion; back them up with references or personal experience.


            Use MathJax to format equations. MathJax reference.


            To learn more, see our tips on writing great answers.




            draft saved


            draft discarded














            StackExchange.ready(
            function () {
            StackExchange.openid.initPostLogin('.new-post-login', 'https%3a%2f%2fmath.stackexchange.com%2fquestions%2f3031094%2fa-proof-that-sqrt2-is-not-a-rational-number%23new-answer', 'question_page');
            }
            );

            Post as a guest















            Required, but never shown





















































            Required, but never shown














            Required, but never shown












            Required, but never shown







            Required, but never shown

































            Required, but never shown














            Required, but never shown












            Required, but never shown







            Required, but never shown







            Popular posts from this blog

            To store a contact into the json file from server.js file using a class in NodeJS

            Redirect URL with Chrome Remote Debugging Android Devices

            Dieringhausen